June 2007 - Sec 1 - Game 1 - Q3

Video Transcript:

0:08
Moving on to Question 3, If the third digit of an acceptable product code is not 0, which
0:16
one of the following must be true?
0:18
Again, we have two possibilities here. We're saying that the third digit is not 0. So drawing
0:27
out two possibilities.
0:31
In Scenario 1 our options for the third spot are 0 and 3. If it's not 0 it's going to be
0:39
3. Since the fifth digit must be greater than the third digit, or the third digit must be
0:48
less than the value of the fifth digit, we know that 4 must be fifth. We know 1 and 2
0:55
would be first and second, which puts 0 in four.
0:59
What about for option 2? In option 2 our options for the third spot are 0 and 1. The stem from
1:08
Question 3 is telling us it's not 0 so it must 1. We know 2 and 4 now are going to be
1:16
first and second. We know that the third digit is less than the value of the fifth digit.
1:24
The only 2 digits remaining are 0 and 3, which means 3 has to go fifth and 0 comes fourth.
1:30
Again, we're looking for what must be true. So Let's take a look:
1:36
(A), the second digit of the product code is 2. That's definitely possible in our
1:40
first scenario, but it doesn't have to be true because in scenario 2 the second digit
1:45
is 4. So (A) is not necessarily true and (A) is out.
1:50
(B), the third digit of the product code is 3. In scenario 1 that's true, but in
1:55
scenario 2 it's 1, so (B) doesn't have to true and (B) is out.
1:59
(C), the fourth digit of the product code is 0. You notice, in both of our scenarios
2:06
here, 0 is the fourth digit. (C) must be true and (C) would be the correct answer.